0
Seien (X,d) ein metrischer Raum und f,g : X nach R stetige Funktionen. Zeigen Sie:
a) Die Funktion h : X nach R, h(x) .= max { fx, g(x) } ist stetig

b) Die Funktion F : X nach R, F(x) := If(x)I ist stetig.

Ich gehe zwar davon aus, das ich bei a) die Dreicksungleichung anwenden muss und den Grenzwert
Epsilon auch ver wenden muss. Aber richtig klar ist mir das nicht. Ich weiß, das die maximum Norm wie folgt aussieht II f II unendlich , ich hoffe, dass das in etwa stimmt.. Der Beweis muss dann mit der Behauptung beginnen und ich muss zeigen, dass das, was ich mit der Dreicksungleichung errechne die Stetigkeit zeigt, und da weiss ich nicht genau, wie ich das anfangen soll. Bei der b) handelt es sich um einen Betrag bei der Funktion, da weiss ich auch nicht wie ich die Stetigkeit zeigen kann.
Diese Frage melden
gefragt

Sonstiger Berufsstatus, Punkte: 139

 

Die Funktion h ist ja so definiert, dass das maximum zwischen den Funktionen f(x) und g(x) stetig ist, ob meine Formulierung ganz exakt ist, weis ich jetzt nicht so genau. Ich muss, so vermute ich zeigen, dass es zwischen beiden Funktionen einen Grenzwert epsilon gibt, dessen Existenz ich mit Hilfe der Dreicksungleichung nachweisen kann. Bei b ist dann ein Betrag gemeint. Hier ist mir der Nachweis der Stetigkeit noch nicht klar.   ─   atideva 04.05.2022 um 09:17

Ich gehe auch davon aus, dass ich das aufgeschrieben habe. Ich habe auch im Internet nochmal danach gesucht. Ich kann das, was ich habe nicht darauf anwenden, bzw. ich weis nicht wie ich es anwenden muss. Was den Grenzwert epsilon betrifft, wurde mir für die Stetigkeit folgendes vorgeschlagen.
aus f(x)<=f(a)+e und g(x)<=g(a)+e folgt max(f(x),g(x))=max(f(a),g(a))+e
aus f(x)>=f(a)-e und g(x)>=g(a)-e folgt max(f(x),g(x))=max(f(a),g(a))-e. mir fehlt hier ungeachtet dem vorliegen von Definitionen sowohl in meinem Skript, wo ich jetzt nicht weis, was ich warum wie anwenden kann. Wie soll das z.B mit deinem Trick konkret gehen? Es gibt ander Dinge die ich gut begreife, mit metrischen Räumen bzw, Funktionen in solchen hab ich es momentan noch nicht so.
  ─   atideva 04.05.2022 um 10:17

Es gibt auch noch die Möglichkeit, dass Definitionen erst mal nicht ganz begriffen werden, weil wie bei mir da eben eine Ladehemmung besteht, die ich versuche zu beheben.   ─   atideva 04.05.2022 um 15:31

Das hilft mir leider im Moment nicht sehr viel weiter. Ich bin ja selbst dabei, im Skript nachzuschauen und im Internet nach möglichen Erklärungen zu suchen. Wenn ich diese Funktion h, deren Definition ich zu Beginn aufgeschrieben habe, erklären müsste, dann würde das ungefähr wie folgt aussehen, da ist eine Funktion , bei der zwei Funktionen einen maximalen Abstand innerhalbe eines Intervalls hat, um einen Grenzwert zu bilden .So oder ähnlich würde ich mich aktuell ausdrücken, und ich vermute mal, dass ich da nicht ganz falsch liege, aber die Sache falsch aufziehe. Richtig erklären kann ich es dann, wenn ich es verstanden habe.   ─   atideva 04.05.2022 um 16:36

Ich habe in diesem Zusammenhang gestern abend zunächst noch ein Video über Stetigkeit von Funktionen in metrischen Räumen gesehen und da wurde dann stur nach Definition umgestellt. und das umgestellte erklärt. Da ahnte ich bereits etwas. Da es nicht dieselbe Funktion wie h:= max { f(x),g(x) } von der ich zeigen sollte, dass sie stetig ist, war mir noch nicht klar, wie ich vorgehen sollte. Ich habe dann heute vor Vorlesungsbeginn eine Studentin gefragt, ob sie mir da weiter helfen konnte. Mir war klar, dass es um Abstände geht, aber mit dem max konnte ich noch nichts anfangen. Sei erklärte mir dann, wie es mit dem epsilon Delta Kriterium funktioniert, und hat mir dann die einzelnen Schritte erklärt. Dann war mir aber auch klar, dass es hier noch locker möglich ist, Fehler einzubauen und es einiges an Übung braucht, um eine Definition, die dann zwar klar ist, entsprechend anzuwenden. Denn selbst wenn das Kriterium graphisch klar ist, ist nicht direkt alles gewonnen und ich erinnere an Situationen wo das allereinfachste zunächst zum Problem wird.   ─   atideva 06.05.2022 um 13:39
Kommentar schreiben
1 Antwort
0
Um Stetigkeit zu beweisen fixieren wir zuerst ein \(x_0\in X\) und ein \(\varepsilon \in \mathbb{R}^+\). Weil \(f\) und \(g\) stetig sind, finden wir ein \(\delta \in \mathbb{R}^+\), so dass für alle \(x \in X\) mit \(d(x,x_0)<\delta\) gilt: \(|f(x)-f(x_0)|<\varepsilon \) und \(|g(x)-g(x_0)|<\varepsilon\) (Epsilon-Kosmetik lasse ich weg). Jetzt musst du \(|h(x)-h(x_0)|\) abschätzen. Tipp zu (b): wie kannst du Betrag mit Maximum ausdrücken?
Diese Antwort melden
geantwortet

Student, Punkte: 10.87K

 

Danke für diese Antwort,, ich glaube schon, das mir das weiterhilft, allerdings habe ich bei metrischen Räumen und in diesem Fall mit dazugehörigen stetigen Funktionen immer noch Ladehemmungen. Wie kann man diese so definierte Funktion sprachlich übersetzten. Ich glaube, wenn mir das klarer ist, dann kann ich Definitionen über metrische Räume.. besser verstehen.   ─   atideva 04.05.2022 um 15:29

Es ist \(h(x)=f(x)\), falls \(f(x)\geq g(x)\) und sonst \(h(x)=g(x)\)   ─   mathejean 04.05.2022 um 17:04

Kommentar schreiben